Diễn Đàn MathScopeDiễn Đàn MathScope
  Diễn Đàn MathScope
Ghi Danh Hỏi/Ðáp Thành Viên Social Groups Lịch Ðánh Dấu Ðã Ðọc

Go Back   Diễn Đàn MathScope > Sơ Cấp > Tài Liệu > Đề Thi > Đề Thi HSG Cấp Tỉnh ở Việt Nam

News & Announcements

Ngoài một số quy định đã được nêu trong phần Quy định của Ghi Danh , mọi người tranh thủ bỏ ra 5 phút để đọc thêm một số Quy định sau để khỏi bị treo nick ở MathScope nhé !

* Nội quy MathScope.Org

* Một số quy định chung !

* Quy định về việc viết bài trong diễn đàn MathScope

* Nếu bạn muốn gia nhập đội ngũ BQT thì vui lòng tham gia tại đây

* Những câu hỏi thường gặp

* Về việc viết bài trong Box Đại học và Sau đại học


Trả lời Gởi Ðề Tài Mới
 
Ðiều Chỉnh Xếp Bài
Old 21-09-2018, 12:06 PM   #16
nguyenhaan2209
+Thành Viên+
 
Tham gia ngày: Jul 2017
Bài gởi: 7
Thanks: 1
Thanked 6 Times in 4 Posts
Câu BĐT đề KHTN:
Với đề bài này ta nghĩ ngay tới phép thế $x=\frac{a}{b+c}, y=\frac{b}{c+a},z=\frac{c}{a+b}$
Biểu thức đã cho trở thành: $A=\sum \frac{ab}{(c+a)(c+b)}+\sqrt{\frac{abc}{(a+b)(b+c)( c+a)}}\sum \sqrt{\frac{a}{b+c}}$
Hay: $1+\sqrt{\frac{abc}{(a+b)(b+c)(c+a)}}(\sum \sqrt{\frac{a}{b+c}}-2\sqrt{\frac{abc}{(a+b)(b+c)(c+a)}})$
Đặt $\sqrt{\frac{abc}{(a+b)(b+c)(c+a)}}=t$
$1.$ Nếu a,b,c không có số nào bằng 0 thì $\sum \sqrt{\frac{a}{b+c}}\geq \sum \frac{a}{\sqrt{a(b+c)}}\geq 2>t$
Nếu có $1$ số $=0$, dễ thấy không đồng thời có $2$ số $=0$ nên WLOG gs $a=0$ thì
$\sum \sqrt{\frac{a}{b+c}}=\sum\sqrt{\frac{b}{c}}\geq 2>t$
Rõ ràng cả $2$ TH đều cho ta $A\geq1$ hay $minA=1$ dấu $"="$ xảy ra khi $(x,y,z)=(1,1,0)$ và các hoán vị
$2$. Ta có BĐT sau: $\sum \sqrt{\frac{2a}{b+c}}\geq 3$
$CM:$ Do vai trò đồng bậc, ta chuẩn hóa $a+b+c=3$
Bằng pp tiếp tuyến, ta CM đc $\sqrt{\frac{2a}{3-a}}\geq 1+\frac{a-1}{4}$
Biến đổi tương đương cái trên thành $(a-1)^2(a+5)\geq0$ hiển nhiên đúng
Áp dụng: $A\leq -2t^2+\frac{3}{\sqrt{2}}t+1$
Kháo sát hàm bậc $2$ trên $[0,\frac{1}{2\sqrt{2}}]$ có $A'=-4t+\frac{3}{\sqrt{2}}>0$ nên hàm đồng biến
Do vậy $A$ đạt cực đại tại $A(\frac{1}{2\sqrt{2}})=\frac{3}{2}$
Dấu $"="$ xảy ra khi $a=b=c=1$ hay $x=y=z=\frac{1}{2}$
------------------------------
Bài $12$ Đại số (Quảng Bình):
Gọi $x_1, x_2, . . . , x_n$ là các nghiệm thực của $P(x)$.
Nếu tồn tại $i ∈ {1, 2, . . . , n}$ sao cho $x_i<y$ thì $P(x_i)>0$ (mâu thuẫn vì $x_i$ là nghiệm của $P(x)$). Vậy ta có $0 < y ≤ x_i (i=1,n)$
Mặt khác ta có $P(x) = (x − x_1) (x − x_2). . .(x − x_n)$ và $n$ chẵn nên
$P(0) = (−1)^nx_1x_2 . . . x_n = x_1x_2 . . . x_n > 0$ (do n là số chẵn)
$P(y) = (y − x_1) (y − x_2). . .(y − x_n)= (x1 − y) (x2 − y). . .(xn − y) ≥ 0$
Điều cần chứng minh trở thành
$[y+\sqrt{(x_1 − y) (x_2 − y). . .(x_n − y)}]^n≤ x_1x_2 . . . x_n$
Áp dụng BĐT Holder ta được: $x_1x_2 . . . x_n = (y + x_1 − y) (y + x_2 − y). . .(y + x_n − y)≥[y+\sqrt{(x_1 − y) (x_2 − y). . .(x_n − y)}]^n$
Đây chính là điều phải chứng minh.
[RIGHT][I][B]Nguồn: MathScope.ORG[/B][/I][/RIGHT]
 

thay đổi nội dung bởi: nguyenhaan2209, 21-09-2018 lúc 12:26 PM Lý do: Tự động gộp bài
nguyenhaan2209 is offline   Trả Lời Với Trích Dẫn
The Following User Says Thank You to nguyenhaan2209 For This Useful Post:
ncthanh (21-09-2018)
Old 21-09-2018, 01:28 PM   #17
nguyenhaan2209
+Thành Viên+
 
Tham gia ngày: Jul 2017
Bài gởi: 7
Thanks: 1
Thanked 6 Times in 4 Posts
Câu $13$ Đại số (Quảng Bình)
$f(x-y)+f(xy)=f(x)-f(y)+f(x)f(y) \forall x\in R$ (1)
Gọi $P(u,v)$ là phép thế $x$ bởi $u$, $y$ bởi $v$ vào (1)
+ $P(x,0): f(0)(f(x)-2)=0$. TH $f(x)=2\forall x\in R$ ta thấy thỏa mãn do đó ta xét $f(0)=0$
+ $P(-1,-1): f(1)=f(-1)^2$ mà $P(0,1): f(-1)=-f(1)$ nên $f(1)=0$ hoặc $1$
$TH1: f(1)=0$ nên $f(-1)=0$
+ $P(x+1,1): f(x)=0$ thử lại ta thấy $f(x)=0 \forall x\in R$ thỏa mãn
TH$2$: $f(1)=1$
+ $P(x+1,1): f(x+1)=f(x)+1$ $(2)$
+ $P(x+1,y): f(x+1-y)+f(xy+y)=f(x+1)-f(y)+f(x+1)f(y)$
Từ $(2)\Rightarrow f(x-y)+1+f(xy+y)=f(x)+1-f(y)+f(x)f(y)+f(y)$
$\Rightarrow f(x-y)+f(xy+y)=f(x)+f(x)f(y)=f(x-y)+f(xy)+f(y)$
$\Rightarrow f(xy+y)=f(xy)+f(y) \Rightarrow f(x+y)=f(x)+f(y)$ $(3)$
+ $P(0,y):f(-y)=-f(y)$ do đó $f(x-y)=f(x)+f(-y)=f(x)-f(y)$
Thay vào $(1)$ thì $f(xy)=f(x)f(y)$ $(4)$
Từ $(3)(4)$ theo kết quả quen thuộc thì $f(x)=x$
Vậy bài toán có $3$ nghiệm hàm là $f(x)\equiv 0, f(x)\equiv 2, f(x)\equiv x \forall x\in R$
[RIGHT][I][B]Nguồn: MathScope.ORG[/B][/I][/RIGHT]
 
nguyenhaan2209 is offline   Trả Lời Với Trích Dẫn
The Following 2 Users Say Thank You to nguyenhaan2209 For This Useful Post:
MATHSCOPE (21-09-2018), ncthanh (21-09-2018)
Old 21-09-2018, 01:38 PM   #18
Le khanhsy
Super Moderator
 
Tham gia ngày: Oct 2017
Bài gởi: 48
Thanks: 52
Thanked 57 Times in 30 Posts
Trích:
Nguyên văn bởi MATHSCOPE View Post
$\boxed{1}$ [Chuyên ĐHSP Hà Nội] Cho $n$ là số nguyên lớn hơn $1$ và $\left\{x_1,\,x_2,\,\ldots,\,x_n\right\}$ là một hoán vị của $\left\{1,\,2,\,\ldots,\,n\right\}$,(tập hợp gồm $n$ số nguyên dương đầu tiên). Chứng minh rằng
\[\sum\limits_{k = 1}^n {k{x_k}\left( {k + {x_k}} \right)} \le \dfrac{{{n^2}{{\left( {n + 1} \right)}^2}}}{2}.\]
Trước tiên chúng ta có đẳng thức quen thuộc sau
\[1^3 + 2^3 + 3^3 + ... + n^3 = \dfrac{ n^2 (n+1)^2}{4} .\]
Theo giả thiết thì và kết hợp đẳng thức trên thì
\[ \sum_{k=1}^nx^3_k+ \sum_{k=1}^nk^3= \dfrac{ n^2 (n+1)^2}{2}.\]
Vì thế bài toán cần chứng minh
\[\sum\limits_{k = 1}^n {k{x_k}\left( {k + {x_k}} \right)} \le \sum_{k=1}^n \left( x^3_k+k^3\right),\]
hay
\[\sum_{k=1}^n \left( x_k-k\right)^2\left( x_k+k\right)\ge 0.\]
Bất đẳng thức trên là luôn đúng trên tập số tự nhiên.
[RIGHT][I][B]Nguồn: MathScope.ORG[/B][/I][/RIGHT]
 
Le khanhsy is offline   Trả Lời Với Trích Dẫn
The Following User Says Thank You to Le khanhsy For This Useful Post:
ncthanh (21-09-2018)
Old 21-09-2018, 02:53 PM   #19
ncthanh
Moderator
 
Tham gia ngày: Oct 2017
Đến từ: THPT Chuyên Bảo Lộc
Bài gởi: 17
Thanks: 51
Thanked 10 Times in 7 Posts
Trích:
Nguyên văn bởi MATHSCOPE View Post

$\boxed{2}$ [Chuyên ĐHSP Hà Nội]Cho tam giác $ABC$ không cân nội tiếp đường tròn $O$, $I$ là tâm đường tròn nội tiếp. Gọi $E$ là giao điểm của $BI$ và $AC$, $F$ là giao điểm của $CI$ và $AB$; $M,\,N$ lần lượt là giao điểm thứ hai của $BI$ và $CI$ và đường tròn $O$. Đường thẳng $BI$ cắt đường tròn ngoại tiếp tam giác $BNF$ tại điểm thứ hai $P$, đường thẳng $CI$ cắt đường tròn ngoại tiếp tam giác $CME$ tại điểm thứ hai $Q$.
  1. Chứng minh rằng tứ giác $EFBQ$ nội tiếp một đường tròn.
  2. Qua $I$ kẻ đường thẳng $\Delta$ vuông góc với $BC$. Chứng minh rằng tâm đường tròn ngoại tiếp tứ giác $EFBQ$ nằm trên $\Delta$.
1. Ta có: $\frac{{IF}}{{IP}} = \frac{{IB}}{{IN}} = \frac{{IC}}{{IM}} = \frac{{IE}}{{IQ}}$, suy ra $IF.IQ = IE.IP$, nên tứ giác $EFPQ$ nội tiếp.

2. Trước tiên ta phát biểu bổ đề sau: " Cho tam giác $ABC$ nhọn nội tiếp đường tròn $(O)$, ngoại tiếp đường tròn (I); đường phân giác của góc $B$ cắt $AC$ tại $E$ và cắt đường tròn (O) lần nữa tại $K$; đường phân giác góc $C$ cắt $AB$ tại $F$ và cắt đường tròn (O) lần nữa tại $L$. Khi đó ta có $IE.BK=IF.CL$"



Chứng minh: Kí hiệu $a$, $b$, $c$ lần lượt là độ dài các cạnh $BC$, $CA$, $AB$; $p$ là nửa chu vi tam giác $ABC$ và $r$ là bán kính đường tròn nội tiếp tam giác $ABC$. Ta có $BK.BE=ca$, đồng thời $\frac{{IE}}{{BE}} = \frac{{\left[ {AIC} \right]}}{{\left[ {ABC} \right]}} = \frac{{\frac{1}{2}br}}{{pr}} = \frac{b}{{a + b + c}}$, nên $BK.IE = \frac{{abc}}{{a + b + c}}$, tương tự $CL.IF = \frac{{abc}}{{a + b + c}}$, suy ra $IE.BK=IF.CL$. Hoàn tất chứng minh.

Trở lại bài toán.



Gọi $K$ là tâm đường tròn ngoại tiếp tứ giác $EFPQ$ . Ta có $K$ nằm trên $\Delta $ nếu và chỉ nếu $IK$ vuông góc với $BC$, hay:\[I{B^2} - I{C^2} = K{B^2} - K{C^2} \qquad(1).\]
Điều này tương đương với: \[ IB.BP + IB.IP - IC.CQ - IC.IQ = BP.BE - CQ.CF.\]
Hay: \[\begin{array}{l}
{\rm{ }}IB.IP - IC.IQ = BP.IE - CQ.IF\\
\Leftrightarrow IN.IF - IE.IM = BP.IE - CQ.IF\\
\Leftrightarrow IE.\left( {BP + IM} \right) = IF.\left( {CQ + IN} \right)\\
\Leftrightarrow IE.\left( {BM - PI} \right) = IF.\left( {CN - IQ} \right)\\
\Leftrightarrow IE.BM = IF.CN. \qquad(2)
\end{array}\]
Tới đây, áp dụng bổ đề vừa chứng minh ta có ngay đẳng thức $(2)$ đúng, tức là đẳng thức $(1)$ đúng, do đó $K$ thuộc $\Delta$.
[RIGHT][I][B]Nguồn: MathScope.ORG[/B][/I][/RIGHT]
 
Hình Kèm Theo
Kiểu File : png bổ đề.PNG (34.6 KB, 124 lần tải)
Kiểu File : png bài làm.PNG (30.6 KB, 123 lần tải)
ncthanh is offline   Trả Lời Với Trích Dẫn
The Following 2 Users Say Thank You to ncthanh For This Useful Post:
Le khanhsy (25-09-2018), nguyenhaan2209 (21-09-2018)
Old 21-09-2018, 04:11 PM   #20
nguyenhaan2209
+Thành Viên+
 
Tham gia ngày: Jul 2017
Bài gởi: 7
Thanks: 1
Thanked 6 Times in 4 Posts
Hình học
$1$. Chú ý tính chất điểm $Kosnita$ ($U$ là tâm $(OBC)$ thì $AU,BP,CQ$ đồng quy). Đoạn sau sử dụng hàng điểm điều hòa và chú ý qua phép đối xứng trục là pg góc $A$ bảo toàn tỉ số kép nên $AX,AY$ đẳng giác
$2$. Câu $a$ biến đổi góc đơn giản. Câu $b$ sử dụng định lí $4$ điểm ĐPCM tương đương với $SB^2-SC^2=IB^2-IC^2$ sau cùng cần cm $IE/IF=BN/CM$ điều này đúng dựa theo định lí $sin$ (lồng $IB, IC$ vào) và ta có ĐPCM
$3$.
$a) OJ.OK=ON.OM=OB.OD$ nên $BJDK$ nội tiếp
$b)$ $OI.OK=1/2R.2R=OA^2$ nên $OAI=OKA=KAB$ nên $AK$, $AI$ đẳng giác góc $BAC$ mà $AI$ là đường kính $(APQ)$ nên $AK$ là đg cao tức $AK$ vg $PQ$
$4$.
$a)$ Theo kết quả quen thuộc thì $BXYC$ nội tiếp mà $XY/BC=XY/2R=sinXBY=sinEFI=sinA/2$ luôn cố định do $A$ const
$b)$ Kẻ đgt qua $N//BC$ ta có $IEG$~$IFH$ do $EIG=END=HNF=HIF$ nên $IH/IG=IE/IF=1$ tức $N$ là tđ $HG$ suy ra $AN$ đi qua tđ $M$ của $BC$
$NX/NY=IX/IY.cosC/2:cosB/2=sinC/2cosC/2:sinB/2cosB/2=sinC/sinB=AC/AB$
$5.$
$a)$$DH.DA=DB.DC=DG.DT$ (Maclaurin) nên $H$ là trực tâm $ATG$ hay $GH$ vg $AT$
$b)$Gọi $IJ$ cắt $(O)$ tại $S$, $GS$ cắt $(O)$ tại $L$.
Ta có: $LC/LB=GC/GB.SB/SC$ (bổ đề quen thuộc) =$GC/GB.IC/IB.JB/JC$ (bổ đề cát tuyến)
$=EC/EA.FA/FB.IC/IB.JB/JC$ (định lí $Menelaus$)
$=(AC/AB)^2.DC/DB.sinPAC/sinPAB.JB/JD.JD/JC$ (định lí $sin$)
$=(AC/AB)^2.DC/DB.AB/AC.PC/PB.BD/PC.BP/DC$ ($JB/JD=JP/JC$) $=AC/AB$
Từ đó theo bổ đề cát tuyến ta có $AL$ đi qua tđ $T$ của $BC$, từ đó rõ ràng thấy $S$ cố định hay $IJ$ luôn đi qua điểm $S$ cố định
[RIGHT][I][B]Nguồn: MathScope.ORG[/B][/I][/RIGHT]
 

thay đổi nội dung bởi: nguyenhaan2209, 21-09-2018 lúc 04:14 PM
nguyenhaan2209 is offline   Trả Lời Với Trích Dẫn
The Following User Says Thank You to nguyenhaan2209 For This Useful Post:
ncthanh (21-09-2018)
Old 21-09-2018, 08:39 PM   #21
Nguyenhuyen_AG
+Thành Viên+
 
Nguyenhuyen_AG's Avatar
 
Tham gia ngày: Apr 2010
Bài gởi: 300
Thanks: 35
Thanked 307 Times in 151 Posts
Trích:
Nguyên văn bởi MATHSCOPE View Post
$\boxed{3}$ [Lạng Sơn] Cho $a,\,b,\,c$ là các số thực dương. Chứng minh rằng \[{\left( {\frac{a}{b} + \frac{b}{c} + \frac{c}{a}} \right)^2} \ge \left( {a + b + c} \right)\left( {\frac{1}{a} + \frac{1}{b} + \frac{1}{c}} \right).\]
Câu này là đề Iran 2005.

Giả sử $a = \max\{a,b,c\}.$ Ta xét hai trường hợp sau:

1. Nếu $b \geqslant c,$ áp dụng bất đẳng thức AM-GM
\[4(a+b+c)\left ( \frac{1}{a}+\frac{1}{b}+\frac{1}{c} \right ) \leqslant \left [\frac{a+b+c}{b}+b\left (\frac{1}{a}+\frac{1}{b}+\frac{1}{c} \right ) \right ]^2.\]
Ta quy bài toán về chứng minh
\[2\left ( \frac{a}{b}+\frac{b}{c}+\frac{c}{a} \right ) \geqslant \frac{a+b+c}{b}+b\left (\frac{1}{a}+\frac{1}{b}+\frac{1}{c} \right ),\]
thu gọn thành
\[\frac{(a-c)\left[b(b-c)+c(a-b)\right]}{abc} \geqslant 0.\]
Hiển nhiên đúng.

2. Nếu $b \leqslant c,$ đánh giá tương tự như trên
\[4(a+b+c)\left ( \frac{1}{a}+\frac{1}{b}+\frac{1}{c} \right ) \leqslant \left [\frac{a+b+c}{c}+c\left (\frac{1}{a}+\frac{1}{b}+\frac{1}{c} \right ) \right ]^2,\]
ta đưa bài toán về chứng minh
\[2\left ( \frac{a}{b}+\frac{b}{c}+\frac{c}{a} \right ) \geqslant \frac{a+b+c}{c}+c\left (\frac{1}{a}+\frac{1}{b}+\frac{1}{c} \right ),\]
tương đương với
\[\frac{(a-b)(2ca-ab-c^2)}{abc} \geqslant 0.\]
\[\frac{(a-b)\left[c(a-c)+a(c-b)\right]}{abc} \geqslant 0.\]
Hiển nhiên đúng.

Trong mọi trường hợp bất đẳng thức đều được chứng minh.

Ngoài ra thì
\[{\left( {\frac{a}{b} + \frac{b}{c} + \frac{c}{a}} \right)^2} \-\left( {a + b + c} \right)\left( {\frac{1}{a} + \frac{1}{b} + \frac{1}{c}} \right) = \sum \frac{(a+b)(a-b)^2}{ab^2} \geqslant 0.\]
------------------------------
Trích:
Nguyên văn bởi Le khanhsy View Post
Ngoài ra chúng ta cũng có thể giải bằng thuần Cauchy-Schwarz một cách dễ dàng
Bài này có thể giải thuần túy Cauchy-Schwarz bằng 2 cách.
[RIGHT][I][B]Nguồn: MathScope.ORG[/B][/I][/RIGHT]
 
__________________
Nguyen Van Huyen
Ho Chi Minh City University of Transport

thay đổi nội dung bởi: Nguyenhuyen_AG, 21-09-2018 lúc 09:17 PM Lý do: Tự động gộp bài
Nguyenhuyen_AG is offline   Trả Lời Với Trích Dẫn
The Following User Says Thank You to Nguyenhuyen_AG For This Useful Post:
ncthanh (21-09-2018)
Old 21-09-2018, 11:13 PM   #22
ncthanh
Moderator
 
Tham gia ngày: Oct 2017
Đến từ: THPT Chuyên Bảo Lộc
Bài gởi: 17
Thanks: 51
Thanked 10 Times in 7 Posts
Trích:
Nguyên văn bởi MATHSCOPE View Post

$\boxed{11}$ [Phú Thọ] Cho dãy số thực $(x_n)_{n\ge 0}$ thỏa mãn đồng thời các điều kiện sau:
  1. $x_n=0$ khi và chỉ khi $n=0$.
  2. ${x_{n + 1}} = x_{\left\lfloor {\frac{{n + 3}}{2}} \right\rfloor }^2 + {( - 1)^n}x_{\left\lfloor {\frac{n}{2}} \right\rfloor }^2$ với mọi $n\ge 0$.
Chứng minh rằng với mọi số nguyên dương $n$, nếu $x_n$ là số nguyên tố thì $n$ là số nguyên tố hoặc $n$ không có ước nguyên tố lẻ.
Dễ thấy ${x_1} = {x_2} = 1$, ${x_3} = 2$.

Ta có: \[\begin{array}{l}
{x_{2n + 1}} = x_{n + 1}^2 + x_n^2,\forall n \ge 0\\
{x_{2n + 2}} = x_{n + 2}^2 - x_n^2,\forall n \ge 0
\end{array}\]
nên ${x_{2n + 1}} + {x_{2n + 2}} = x_{n + 1}^2 + x_{n + 2}^2 = {x_{2n + 3}},\forall n \ge 0, \qquad(1).$

Bằng quy nạp, ta sẽ chứng minh \[{x_{2n}} = {x_{2n - 1}} + {x_{2n - 2}}, \forall n \ge 1, \qquad(2)\]

Với $n=1$ dễ thấy mệnh đề đúng.

Giả sử mệnh đề đúng đến $n = k \ge 1$, tức là ${x_{2i}} = {x_{2i - 1}} + {x_{2i - 2}},\forall i = \overline {1,k} $. Ta cần chứng minh mệnh đề cũng đúng với $n=k+1$. Thật vậy, ta có: \[{x_{2k + 2}} - {x_{2k}} = \left( {x_{k + 2}^2 - x_k^2} \right) - \left( {x_{k + 1}^2 - x_{k - 1}^2} \right) = {\left( {{x_{k + 1}} + {x_k}} \right)^2} - {\left( {{x_{k + 1}} - {x_k}} \right)^2} - \left( {x_{k + 1}^2 + x_k^2} \right) = x_{k + 1}^2 + x_k^2 = {x_{2k + 1}}.\]
Suy ra ${x_{2k + 2}} = {x_{2k + 1}} + {x_{2k}}$, tức là mệnh đề cũng đúng với $n=k+1$. Do đó $(2)$ được chứng minh.

Từ $(1)$ và $(2)$ ta thu được ${x_{n + 2}} = {x_{n + 1}} + {x_n},\forall n \ge 0.$, đồng thời, để ý rằng ${x_0} = 0$ và ${x_1} = 1$ nên $\left( {{x_n}} \right)$ là dãy Fibonacci.

Từ đó, áp dụng tính chất quen thuộc của dãy Fibonacci: "${F_m}\mid {F_n}$ khi và chỉ khi $m\mid n$" ta có ngay điều cần chứng minh. Bởi nếu không, viết $n=qm$, với $m$ là số nguyên tố lẻ và $q$ là số tự nhiên lớn hơn 1, lúc đó ${x_m}\mid {x_{qm}} = {x_n}$, mà $\left( {{x_n}} \right)$ là dãy tăng nên ${x_m} \ge {x_3} = 2$, dẫn tới ${x_n}$ là hợp số. Vô lý!!
[RIGHT][I][B]Nguồn: MathScope.ORG[/B][/I][/RIGHT]
 
ncthanh is offline   Trả Lời Với Trích Dẫn
The Following User Says Thank You to ncthanh For This Useful Post:
MATHSCOPE (22-09-2018)
Old 22-09-2018, 03:27 PM   #23
Thụy An
+Thành Viên+

 
Tham gia ngày: Oct 2017
Bài gởi: 93
Thanks: 1
Thanked 68 Times in 45 Posts
Trích:
Nguyên văn bởi MATHSCOPE View Post
$\boxed{14}$ [Đà Nẵng] Cho $p$ là một số nguyên tố lẻ, số nguyên dương n được gọi là "tốt" nếu tồn tại đa thức $P(x)$ với hệ số nguyên, có bậc bằng $p$ và hệ số bậc cao nhất bằng 1 sao cho $n$ là ước số của $P(k)$ với mọi số nguyên $k$. Một số nguyên dương mà không phải là số tốt được gọi là số "xấu". Chứng minh rằng:
  1. $p$ là số tốt,
  2. $p^2$ là số xấu.
  1. Xét $P_p(x)=x^p-x$, theo định lý Fermat bé ta có\[p\mid P_p(k)\quad\forall\,k\in\mathbb Z.\]Do vậy, $p$ là số tốt.

  2. Nếu $p^2$ là số "tốt", ta giả sử $P(x)$ là đa thức monic có bậc $p$ và thỏa mãn\[p^2\mid P(k)\quad\forall\,k\in\mathbb Z.\]Khi đó kéo theo $p\mid P(k)\;\forall\,k\in\mathbb Z$, cho nên $P(x)=x^p-x+pQ(x)$ với $Q\in\mathbb Z[x]$, ta có $p^2\mid P(0)$ nên $p\mid Q(0)$. Lại vì $Q\in\mathbb Z[x]$ nên $p\mid Q(p)$ từ đó dẫn đến mâu thuẫn là\[0 \equiv P\left( p \right) \equiv {p^p} - p + pQ\left( p \right) \equiv - p\pmod{p^2}.\] Vậy, $p^2$ là số "xấu".

[RIGHT][I][B]Nguồn: MathScope.ORG[/B][/I][/RIGHT]
 
Thụy An is offline   Trả Lời Với Trích Dẫn
The Following User Says Thank You to Thụy An For This Useful Post:
ncthanh (22-09-2018)
Old 25-09-2018, 12:31 PM   #24
nguyenhaan2209
+Thành Viên+
 
Tham gia ngày: Jul 2017
Bài gởi: 7
Thanks: 1
Thanked 6 Times in 4 Posts
Bài $16$ (Hưng Yên):
Ta giải quyết bài toán tổng quát khi thay $2018$ bởi $n$
Ta CM mệnh đề bài toán bằng phương pháp quy nạp
Xét đa thức $f_n(x)=\prod (x\pm \sqrt{a_1}x\pm...x\pm \sqrt{a_{n}})$
Đề bài tương đương với việc cm $f_n(\sqrt{a_{n+1}})f_n(-\sqrt{a_{n+1}})$ là scp
$1$. $f_n(x) \in Z_{[x]}: f_n(x)=f_{n-1}(x-\sqrt{a_n})f_{n-1}(x+\sqrt{a_n})=A^2-B^2a_n \in Z \forall x \in Z$
Bằng công thức nội suy Lagrange ta dễ dàng có $f_n(x) \in Z_{[x]}$
$2$. Dễ thấy $f_n(x)$ chứa $2^n$ thừa số mà các thừa số có thể ghép cặp với nhau nên $f_n(x)$ là hàm chẵn
Từ $(1)(2)$: $f_n(x)=g_n(x^2)$ với $g_n(x) \in Z_{[x]}$ nên $f_n(\sqrt{a_{n+1}})f_n(-\sqrt{a_{n+1}})=g_n^2(a_{n+1})$ là SCP

Bài $17$ (Hưng Yên):
Ta có $2$ nhận xét sau
$1$. Không có $2$ số lẻ nào kề nhau: Giả sử ngược lại tồn tại $2$ số lẻ kề nhau khi đó dựa vào $ii)$ dễ có tất cả các số trên bảng lẻ vô lí do trái với $i)$
$2$. Không có $2$ số chẵn nào kề nhau: CMTT như trên
Xét khoảng cách giữa $2$ số lẻ $x_i, x_j$ liên tiếp:
Do $(1)$ nên $x_i-x_j \geq 2$ mặt khác nếu $x_i-x_j=2$ thì số nằm giữa là số lẻ vô lí
Nếu $x_i-x_j>3$ thì giữa chúng có $>3$ số chẵn cũng vô lí theo $(2)$
Vậy rõ ràng $2$ số lẻ liên tiếp cách nhau bởi $2$ số chẵn
Ta thiết lập ánh xạ sao cho mỗi số lẻ đi kèm bởi $2$ số chẵn ngay sau nó dễ thấy chúng tạo thành $1$ chu trình đi hết đa giác nên số đỉnh của đa giác phải chia hết cho $3$ và ta có ĐPCM
[RIGHT][I][B]Nguồn: MathScope.ORG[/B][/I][/RIGHT]
 

thay đổi nội dung bởi: nguyenhaan2209, 25-09-2018 lúc 12:48 PM Lý do: Tự động gộp bài
nguyenhaan2209 is offline   Trả Lời Với Trích Dẫn
Old 25-09-2018, 05:00 PM   #25
Thụy An
+Thành Viên+

 
Tham gia ngày: Oct 2017
Bài gởi: 93
Thanks: 1
Thanked 68 Times in 45 Posts
Trích:
Nguyên văn bởi nguyenhaan2209 View Post
Dễ thấy $f_n(x)$ chứa $2^n$ thừa số
Cái dễ thấy này sai!!
[RIGHT][I][B]Nguồn: MathScope.ORG[/B][/I][/RIGHT]
 
Thụy An is offline   Trả Lời Với Trích Dẫn
Old 26-09-2018, 10:15 AM   #26
Ngocanh9x
+Thành Viên+
 
Tham gia ngày: Dec 2017
Bài gởi: 11
Thanks: 2
Thanked 1 Time in 1 Post
Trích:
Nguyên văn bởi nguyenhaan2209 View Post
Bài $16$ (Hưng Yên):
Với $a_1=a_2=\ldots =a_{2017}=1$ và $a_{2018}=2018^2$, ta có\[S_{2018}=\{-4035,\,-4033,\,\ldots,\,-1,\,1,\,\ldots ,\,4033,\,4035\}.\]Từ đó \[\prod_{e\in S_{2018}}e=-\left( \frac{4035!}{2^{2017}.2017!} \right)^2<0. \]Điều này chứng tỏ đề sai!
[RIGHT][I][B]Nguồn: MathScope.ORG[/B][/I][/RIGHT]
 
Ngocanh9x is offline   Trả Lời Với Trích Dẫn
Old 27-09-2018, 01:16 AM   #27
kenzie
+Thành Viên+
 
Tham gia ngày: May 2017
Bài gởi: 19
Thanks: 2
Thanked 3 Times in 3 Posts
Trích:
Nguyên văn bởi MATHSCOPE View Post
$\boxed{18}$ [Đồng Nai] Tìm số nguyên tố $p$ sao cho phương trình nghiệm nguyên sau có nghiệm\[4x^2+12xy+13y^2=p.\]
Phương trình tương đương với\[(2x+3y)^2+(2y)^2=p.\]Tới đây đơn giản, vì $p=2$ thì không có nghiệm, còn với $p$ lẻ thì phương trình đó có nghiệm khi và chỉ khi $p\equiv 1\pmod 4.$
[RIGHT][I][B]Nguồn: MathScope.ORG[/B][/I][/RIGHT]
 
kenzie is offline   Trả Lời Với Trích Dẫn
Old 29-09-2018, 03:03 AM   #28
Thụy An
+Thành Viên+

 
Tham gia ngày: Oct 2017
Bài gởi: 93
Thanks: 1
Thanked 68 Times in 45 Posts
Trích:
Nguyên văn bởi MATHSCOPE View Post
$\boxed{24}$ [Hà Nội] Xét các số hứu tỉ dương $x_1,\,x_2,\,\ldots,\, x_n$ thỏa mãn ${x_1} + \dfrac{1}{p_1},\,{x_2} + \dfrac{1}{{{p_2}}},\, \ldots, \,{x_n} + \dfrac{1}{p_n}$ là các số nguyên dương (với ${p_i} = \dfrac{{{x_1}{x_2} \ldots {x_n}}}{{{x_i}}},\; \forall i=\overline{1,\,n}$ ).
  1. Chứng minh rằng $x_1x_2\ldots x_n=1.$

  2. Có bao nhiêu bộ số $\left(x_1,\,x_2,\,\ldots,\, x_n\right)$ thỏa mãn đề bài.
Bài toán không đúng nếu $n=1$, ta xét với $n>1$.

Đặt ${{x_1}{x_2} \ldots {x_n}}=P$, ta có $x_i+\dfrac{x_i}{P}\in\mathbb Z,\; \forall i=\overline{1,\,n}$, tức là với số nguyên tố $p$ bất kỳ ta có\[\begin{align}0 \le {v_p}\left( {{x_i} + \frac{{{x_i}}}{P}} \right) = {v_p}\left( {{x_i}} \right) + {v_p}\left( {1 + P} \right) - {v_p}\left( P \right)\quad \forall i=\overline{1,\,n},\quad (1).\end{align}\]Lấy tổng lại và để ý $v_p(P)=\sum\limits_{1\le i\le n}v_p\left (x_i \right )$, ta có\[0 \le n{v_p}\left( {1 + P} \right)-(n-1){v_p}\left( P \right) ,\quad (2).\]
  1. Xét hai trường hợp sau:
    1. Nếu $v_p(P)>0=v_p(1)$, ta có $v_p(1+P)=v_p(1)=0$ từ $(2)$ có điều mâu thuẫn với trường hợp đang xét là\[0 \le - \left( {n - 1} \right){v_p}\left( P \right).\]
    2. Nếu $v_p(P)<0=v_p(1)$, ta có $v_p(1+P)=v_p(P)$ nên từ $(2)$ có điều mâu thuẫn với trường hợp đang xét là\[0 \le {v_p}\left( P \right).\]
    Tóm lại $v_p(P)=0$ với mọi số nguyên tố $p$, nên $P=1$.

  2. Với mọi số nguyên tố $p>2$, thay $P=1$ vào $(1)$ ta có\[0 \le {v_p}\left( {{x_i}} \right) + {v_p}\left( 2 \right) - {v_p}\left( 1 \right) = {v_p}\left( {{x_i}} \right).\]Nhưng, $\sum\limits_{1\le i\le n}v_p\left( {{x_i}} \right)=v_p(P)=0$ nên kéo theo $v_p\left( {{x_i}} \right)=0$ với mọi số nguyên tố $p$ lẻ, thêm nữa cũng từ $(1)$ ta có\[0 \le {v_2}\left( {{x_i}} \right) + {v_2}\left( 2 \right) + {v_2}\left( 1 \right) = 1 + {v_2}\left( {{x_i}} \right)\quad\forall i=\overline{1,\,n}.\]Từ đó $x_i=2^{-1+k_i}$ với $k_i=1+v_2\left(x_i\right)\in\mathbb N$ đồng thời\[0 = {v_2}\left( P \right) = \sum\limits_{1 \le i \le n} {\left( { - 1 + {k_i}} \right) = \sum\limits_{1 \le i \le n} {{k_i}} } - n.\]Mỗi một bộ $\left(k_1,\,k_2,\,\ldots ,\,k_n\right)\in\mathbb N^n$ thỏa $\sum\limits_{1 \le i \le n} {{k_i}} =n$ sẽ cho ta duy nhất 1 bộ $\left(x_1,\,x_2,\,\ldots ,\,x_n\right)\in\left(\mathbb Q^+\right)^n$ bởi quy tắc tương ứng $x_i=2^{-1+k_i}$. Cho nên theo bài toán chia kẹo, kết quả là $\dbinom{2n-1}{n-1}$.

[RIGHT][I][B]Nguồn: MathScope.ORG[/B][/I][/RIGHT]
 
Thụy An is offline   Trả Lời Với Trích Dẫn
Old 30-09-2018, 01:26 PM   #29
nguyenhaan2209
+Thành Viên+
 
Tham gia ngày: Jul 2017
Bài gởi: 7
Thanks: 1
Thanked 6 Times in 4 Posts
Số học:
Câu $14$: Tổng quát $2018$ bởi $n$ ta giải quyết bài toán như sau
Giả sử $a_1=min(a_i)$. Xét TH $n$ lẻ thì $n+1$ chẵn do đó $a^{n+1}=(a_1-1)^{n+1} (mod a+a_1-1)$. Theo gt ta đặt $a=a_1k$. Chú ý rằng $(a_1-1)^{n+1}+a-1=(a_1-1)^{n+1}-a+a_1k \vdots a_1$
Từ đó kết hợp với $gcd(a_1,a+a_1-1)=1$ nên $a_1(a+a_1-1)|(a_1-1)^{n+1}+a-1$ nên $(a_1-1)^{n+1}+a-1 \geq a_1(a+a_1-1)$
Hay $(a_1-1)^{n+1}+a_1k \geq a_1^2k+a_1^2-a_1$
Chú ý rằng $k \geq a_1^{n-1}$ nên $(a_1-1)^{n+1} \geq a_1(a_1-1)k+a_1^2+a_1-a_1+1>a_1(a_1-1)a_1^{n-1}=a_1^n(a_1-1)$ suy ra $(a_1-1)^{n+1} \geq a_1^n(a_1-1)$ vô lí
TH$2$: $n$ chẵn nên $n+1$ lẻ tương tự ta có $(a+a_1-1)|(a_1-1)^{n+1}-a+1$ mà $n+1$ lẻ nên $(a_1-1)^{n+1}+1 \equiv 0 (mod a_1)$. Do vậy $a_1(a+a_1-1)|(a_1-1)^{n+1}-1$ nên
$(a_1-1)^{n+1}<a$ khi ấy $a-(a_1-1)^{n+1}-1 \geq 0$ mà $a_1(a+a_1-1)>a-(a_1-1)^{n+1}-1$ nên buộc $a-(a_1-1)^{n+1}-1=0$ hay $a=(a_1-1)^{n+1}+1$. Khi đó $((a_1-1)^{n+1}+a_1)|((a_1-1)^{n+1}+1)^{n+1}-(a_1-1)^{n+1}$ hay $(a_1-1)^{n+1}+a_1|a_1^{n+1}-a_1$ vậy $(a_1-1)^{n+1}+a_1|a_1^n-1$ vô lí do $VT>VP$ (cm bằng quy nạp) và ta có ĐPCM

Câu $19$: Áp dụng bổ đề $(4x^2-1)^2$ chia hết cho $4xy-1$ khi và chỉ khi $x=y$ (Vieta jumping-IMO 2007) từ đó thay $x=k, y=3n$ ta có $k=3n$
Câu $20$: a) Ta giải quyết bằng cách chọn $4$ số sao cho không có điểm bất động, bằng xét TH ta được kq là $10$. Mà có $41$ lớp đồng dư module $41$ nên kq là $10^{41}$
$b)$ Sử dụng luật thuận nghịch Gauss và chú ý $41$ có dạng $8k+1$ nên $(2/41)=1$ từ đó ta tìm đc $20$ scp mod $41$ và $20$ số ko cp mod $41$. Ta giải quyết $41$ số đầu nhận giá trị từ $1$ đến $41$, $123$ số sau đơn thuần là đồng dư với $41$ số đầu theo mod $41$. Ta chỉ ra chu trình sau: $(2,6,13,12,11,7,14,28,15,17,19,22,34,29,24,35,27, 26,30,38,36,31,23,37,1,32,8,5,4,10,18,21,33,40,20, 39,9,35,23,16,41)$ thỏa mãn đề bài và câu trả lời là tồn tại (Tính toán khá mệt)
Câu $22$: Xét ước và chặn khoảng của $n$ từ đó tìm đc $n=897$
Câu $23$: Trừ các đa thức cho nhau ta có $a,b+1,c$ chia hết cho $p$
[RIGHT][I][B]Nguồn: MathScope.ORG[/B][/I][/RIGHT]
 

thay đổi nội dung bởi: nguyenhaan2209, 30-09-2018 lúc 01:31 PM
nguyenhaan2209 is offline   Trả Lời Với Trích Dẫn
Old 30-09-2018, 05:07 PM   #30
Thụy An
+Thành Viên+

 
Tham gia ngày: Oct 2017
Bài gởi: 93
Thanks: 1
Thanked 68 Times in 45 Posts
Trích:
Nguyên văn bởi nguyenhaan2209 View Post
$b)$ ... Ta giải quyết $41$ số đầu nhận giá trị từ $1$ đến $41$, $123$ số sau đơn thuần là đồng dư với $41$ số đầu theo mod $41$. Ta chỉ ra chu trình sau: $(2,6,13,12,11,7,14,28,15,17,19,22,34,29,24,35,27, 26,30,38,36,31,23,37,1,32,8,5,4,10,18,21,33,40,20, 39,9,35,23,16,41)$ thỏa mãn đề bài và câu trả lời là tồn tại (Tính toán khá mệt)
Lời giải sai, vì $2+6+13+12+11+7+14+28=93$ và\[\left( {\frac{{93}}{{41}}} \right) = \left( {\frac{{11}}{{41}}} \right) = - 1.\]
Trích:
Nguyên văn bởi MATHSCOPE View Post
$\boxed{20}$ [Sài Gòn] Gọi $S$ là tập hợp các hoán vị của 164 số nguyên dương đầu tiên. Tồn tại hay không một hoán vị $\left(a_1,\,a_2,\,\ldots ,\,a_{164}\right)\in S$, thỏa mãn với mỗi $i\in\{1,\,2,\,\ldots ,\,164\}$ luôn tồn tại $b_i\in\{0,\,1,\,\ldots ,\,40\}$ sao cho $a_1+a_2+\ldots+a_i\equiv b_i^2\pmod{41}$.
Giả sử $r_k$ là số dư của $6^k$ khi chia $41$, do $6$ là căn nguyên thủy mod $41$ và\[6^2+6\equiv 1\pmod{41},\quad 6^{20}+1\equiv 0\pmod{41}.\]Nên ta chia $40$ số nguyên dương đầu tiên làm 10 bộ, mỗi bộ $4$ số ở dạng\[\left( {{r_{2k}},\;{r_{2k - 1}},\;{r_{20 + 2k - 1}},\;{r_{20 + 2k}}} \right),\quad\forall\,k = 1,{\mkern 1mu} 2,{\mkern 1mu} \ldots ,{\mkern 1mu} 10.\]Còn $a_i=i$ nếu $41\mid i$, và $a_{41t+r}=41t+a_r$ với $t=0,\,1,\,2,\,3$ và $1\le r\le 40$.

Để ý rằng, với mỗi $0\le i\le 9,\,0\le t\le 3$ sẽ tồn tại $k\in\mathbb Z^+$ thỏa $$\left( {{a_{41t+4i + 1}} ,\; {a_{41t+4i + 2}} ,\; {a_{41t+4i + 3}} ,\; {a_{41t+4i + 4}}} \right)=\left( 41t+{{r_{2k}},\;41t+{r_{2k - 1}},\;41t+{r_{20 + 2k - 1}},\;41t+{r_{20 + 2k}}} \right).$$ Vì thế nên ta có\[\begin{array}{l}
{a_{41t+4i + 1}} + {a_{41t+4i + 2}} + {a_{41t+4i + 3}} + {a_{41t+4i + 4}} &\equiv {r_{2k}} + {r_{2k - 1}} + {r_{20 + 2k - 1}} + {r_{20 + 2k}}\\
&\equiv {6^{2k}} + {6^{2k - 1}} + {6^{20 + 2k - 1}} + {6^{20 + 2k}}\\
& \equiv \left( {{6^{20}} + 1} \right)\left( {{6^{2k}} + {6^{2k - 1}}} \right)\\
&\equiv 0\pmod{41}.
\end{array}\]Kết hợp $41\mid a_i$ nếu $i\mid 41$, nên nếu $m$ chia $41$ dư $n$ thì có\[S_m={a_1} + {a_2} + \ldots + {a_m} \equiv {a_1} + {a_2} + \ldots + {a_n}\pmod{41}.\]Giả sử $n$ chia $4$ dư $r$, ta xét
  1. Nếu $r=0$, thì $S_m\equiv 0\pmod{41}$.
  2. Nếu $r=1$, thì tồn tại $i,\,k\in\mathbb Z^+$ thỏa mãn\[{S_m} \equiv {a_{4i + 1}} \equiv {r_{2k}} \equiv {\left( {{6^k}} \right)^2} \equiv {\left( {{r_k}} \right)^2}\quad \left( {\bmod 41} \right).\]
  3. Nếu $r=2$, thì tồn tại $i,\,k\in\mathbb Z^+$ thỏa mãn\[\begin{array}{l}
    {S_m} &\equiv {a_{4i + 1}} + {a_{4i + 2}} \equiv {r_{2k}} + {r_{2k - 1}}\\
    &\equiv {6^{2k}} + {6^{2k - 1}} \equiv {6^{2k - 2}}\left( {{6^2} + 6} \right)\\
    &\equiv {\left( {{r_{k - 1}}} \right)^2}\quad \left( {{\mkern 1mu} \,\bmod \,{\mkern 1mu} 41} \right).
    \end{array}\]
  4. Nếu $r=3$, thì tồn tại $i,\,k\in\mathbb Z^+$ thỏa mãn\[\begin{array}{l}
    {S_m} &\equiv {a_{4i + 1}} + {a_{4i + 2}} + {a_{4i + 3}} \\
    &\equiv {r_{2k}} + {r_{2k - 1}} + {r_{20 + 2k - 1}} \equiv {6^{2k}} + {6^{2k - 1}} + {6^{20 + 2k - 1}}\\
    &\equiv {6^{2k}} + {6^{2k - 1}}\left( {{6^{20}} + 1} \right)\\
    &\equiv {\left( {{r_k}} \right)^2}\quad \left( {\bmod 41} \right).
    \end{array}\]
Tóm lại là tồn tại hoán vị như yêu cầu.
[RIGHT][I][B]Nguồn: MathScope.ORG[/B][/I][/RIGHT]
 
Thụy An is offline   Trả Lời Với Trích Dẫn
The Following 2 Users Say Thank You to Thụy An For This Useful Post:
huynhcongbang (03-10-2018), Le khanhsy (28-10-2018)
Trả lời Gởi Ðề Tài Mới

Bookmarks

Ðiều Chỉnh
Xếp Bài

Quuyền Hạn Của Bạn
You may not post new threads
You may not post replies
You may not post attachments
You may not edit your posts

BB code is Mở
Smilies đang Mở
[IMG] đang Mở
HTML đang Tắt

Chuyển đến


Múi giờ GMT. Hiện tại là 07:20 PM.


Powered by: vBulletin Copyright ©2000-2024, Jelsoft Enterprises Ltd.
Inactive Reminders By mathscope.org
[page compression: 122.70 k/138.82 k (11.61%)]